Geometry At It's Best

Geometry Level 5

In the Fig. A B C \triangle ABC is such that D & E D \ \& \ E lie on A B AB and F & G F \ \& \ G lie on B C BC .

  • D D and E E are points of trisection of A B AB .

  • F F divides B C BC in the ratio 1 : 3 1:3 and G G divides B C BC in the ratio 3 : 1 3:1

A F , A G , C D & C E AF , AG , CD \ \& \ CE are drawn such that they intersect to form P Q S R \Box PQSR

A ( A B C ) A ( P Q S R ) = m n \dfrac{A(\triangle ABC)}{A(\Box PQSR)}=\dfrac{m}{n}

Find m n m-n


Details and Assumptions :-

  • g c d ( m , n ) = 1 gcd(m,n)=1


The answer is 401.

This section requires Javascript.
You are seeing this because something didn't load right. We suggest you, (a) try refreshing the page, (b) enabling javascript if it is disabled on your browser and, finally, (c) loading the non-javascript version of this page . We're sorry about the hassle.

2 solutions

Anatoliy Razin
Dec 18, 2014

We use 4 different ways to set triangle vertices with masses. On the picture each center of mass marked as O.

Nice approach using center of masses. Can you add more details to explain this solution?

Calvin Lin Staff - 6 years, 5 months ago

Log in to reply

No problem. I posted a new article which will explain my solution as well as good approach to many other similar problems. Also I posted a note with the link to the article. I am relatively new to brilliant but hope I did everything right.

see the note with the link to the article

Anatoliy Razin - 6 years, 5 months ago
Kartik Sharma
Dec 25, 2014

Well, a r ( P Q R S ) = a r ( A F G ) a r ( A P R ) a r ( Q S G F ) ar(PQRS) = ar(AFG) - ar(APR) - ar(QSGF)

a r ( P Q R S ) = a r ( A F G ) a r ( A P R ) a r ( S G B D ) + a r ( D Q F B ) ar(PQRS) = ar(AFG) - ar(APR) - ar(SGBD) + ar(DQFB) ----4

We will adopt the same strategy for finding the answer.

We know that

area of any triangle = 1 2 a b s i n θ \frac{1}{2}absin\theta

hence,

a r ( A F G ) = 1 2 a r ( A B C ) ar(AFG) = \frac{1}{2}*ar(ABC)

In ABF,

using Menealus' theorem, B C C F Q F A Q A D D B = 1 \frac{BC}{CF}\frac{QF}{AQ}\frac{AD}{DB} = -1

Hence, Q F A Q = 3 8 , A F A Q = 11 8 \frac{QF}{AQ} = \frac{3}{8}, \frac{AF}{AQ} = \frac{11}{8}

a r ( A F B ) a r ( A D Q ) = A B . A F A D . A Q = 33 16 \frac{ar(AFB)}{ar(ADQ)} = \frac{AB.AF}{AD.AQ} = \frac{33}{16}

a r ( A F B ) a r ( D Q F B ) = 33 17 \frac{ar(AFB)}{ar(DQFB)} = \frac{33}{17}

Also, a r ( A B F ) = 1 4 a r ( A B C ) ar(ABF) = \frac{1}{4}*ar(ABC)

a r ( A B C ) a r ( D Q F B ) = 132 17 \frac{ar(ABC)}{ar(DQFB)} = \frac{132}{17}

a r ( D Q F B ) a r ( A B C ) = 17 132 \frac{ar(DQFB)}{ar(ABC)} = \frac{17}{132} ----1

In BDC,

using Menealus' theorem, A B A D D S S C G C B G = 1 \frac{AB}{AD}\frac{DS}{SC}\frac{GC}{BG} = -1

hence, D S S C = 2 , D C S C = 3 \frac{DS}{SC} = 2, \frac{DC}{SC} = 3

a r ( S G C ) a r ( D B C ) = 12 \frac{ar(SGC)}{ar(DBC)} = 12

a r ( D S G B ) a r ( D B C ) = 11 12 \frac{ar(DSGB)}{ar(DBC)} = \frac{11}{12}

Also, a r ( D B C ) = 1 3 a r ( A B C ) ar(DBC) = \frac{1}{3}*ar(ABC)

a r ( D S G B ) a r ( A B C ) = 11 36 \frac{ar(DSGB)}{ar(ABC)} = \frac{11}{36} -----2

Similarly, a r ( A P R ) = 4 15 a r ( A F G ) ar(APR) = \frac{4}{15}ar(AFG)

a r ( A F G ) a r ( A P R ) = 11 30 a r ( A B C ) ar(AFG) - ar(APR) = \frac{11}{30}ar(ABC) -----3

Substituting 1, 2, 3 in 4,

a r ( P Q R S ) = 94 495 a r ( A B C ) ar(PQRS) = \frac{94}{495}ar(ABC)

a r ( A B C ) a r ( P Q R S ) = 495 94 \frac{ar(ABC)}{ar(PQRS)} = \frac{495}{94}

Nicer approach - finding ar(APR) and ar(AQS) and then subtracting. Both the areas can be found using the same method as used in the solution.

Kartik Sharma - 6 years, 5 months ago

Log in to reply

Isn't that Menelaus' theorem instead of Appolonius?

Ayush Garg - 5 years, 7 months ago

Log in to reply

Yeah! Sorry. Edited!

Kartik Sharma - 5 years, 7 months ago

0 pending reports

×

Problem Loading...

Note Loading...

Set Loading...